Encontrar la energía almacenada en una capa esférica, pero la integral diverge

Estoy tratando de encontrar la energía almacenada al ensamblar una capa esférica (indicada por S ) uniformados repartidos de carga total q y radio R . Para hacerlo, quiero usar la fórmula:

W = 1 2 ϵ 0 S σ V d a

El problema viene cuando trato de calcular V . Usando la ley de Gauss para encontrar mi y luego V , es fácil ver que V = q 4 π ϵ 0 R en la superficie de la esfera. Sin embargo, cuando trato de calcular V usando un método integral, la integral diverge. La fórmula para V es

V = 1 4 π ϵ 0 S σ | r r | d a

dónde r es la ubicación de ρ y r es la posición de los otros cargos que afecta ρ . Se puede ver que esta integral diverge, lo cual tiene sentido porque estamos asumiendo una distribución de carga continua, lo que significa | r r | 0 .

no entiendo porque es esto? Usando estos métodos integrales, aún deberíamos llegar a la misma respuesta que el método de la ley de Gauss. ¿Podría alguien explicar por qué esto no funciona o una forma de solucionar este problema?

Respuestas (2)

La integral no diverge. Aunque |r −r′|→0, d a = pecado ( θ ) d θ d ϕ enfoques 0 también. Usando coordenadas esféricas con el punto en el que se está evaluando el potencial ubicado en el polo norte obtenemos:

0 2 π 0 π pecado ( θ ) d θ d ϕ 2 2 porque ( θ ) = 4 π

¡Tienes razón! Mi error, no estoy del todo seguro de cómo lo arruiné. Me olvidé del signo de la raíz cuadrada cuando uso la ley de los cosenos en |rr'|. Pensé en borrar toda esta publicación, pero tal vez sea útil para otros que también se olvidan accidentalmente. Gracias por la respuesta.

Sabemos que el campo eléctrico dentro de una capa esférica con carga uniforme es cero, porque todas las cargas se encuentran en la superficie exterior. es decir,

mi i norte = 0
y
mi o tu t = q 4 π ϵ o r 2 r ^

La energía electrostática almacenada en la capa esférica es:

tu = 1 2 ϵ o V mi 2 d τ

tu = 1 2 ϵ o V mi i norte 2 d τ 0 + 1 2 ϵ o V mi o tu t 2 d τ

tu = 1 2 ϵ o V mi o tu t 2 d τ

= 1 2 ϵ o V ( q 4 π ϵ o r 2 ) 2 4 π r 2 d r

= 1 2 ϵ o ( q 4 π ϵ o ) 2 × 4 π R 1 r 2 d r

tu = q 2 8 π ϵ o R